Put the quadratic into vertex form and state the coordinates of the vertex.
y = x2 – 10x + 9
What is the vertex form?

Answers

Answer 1

Answer: y=(x-5)^2-16

Answer 2

Answer:

Step-by-step explanation:

First, we need to use the formula -b/2a.

We get 10/2 which equals 5.

Now we have our x-coordinate. Now we need to find out y-coordinate. We have to plug 5 back in as x to get the y-variable.

5^2 - 10(5) + 9

25 - 50 + 9

34 - 50

-16

Now that we have our x and y coordinates, we can make our vertex.

(5,-16)

Finally, we need to put this into vertex form. We see that the vertex form is y = a(x-h)^2 + k. We plug the x variable into the h value and the y variable into the k value. We don't need the a variable because we are only looking for the vertex and the vertex form.

y = (x-5)^2 - 16

Walah! There is our answer!


Related Questions

Determine if the specified linear transformation is ​(a​) ​one-to-one and ​(b​) onto. Justify your answer. ​T: ​, ​T(​)​(​,​), ​T(​)​(​,​), and ​T(​)​(​,​), where ​, ​, are the columns of the 33 identity matrix. a. Is the linear transformation​ one-to-one? A. T is​ one-to-one because ​T(x​)0 has only the trivial solution. B. T is not​ one-to-one because the columns of the standard matrix A are linearly independent. C. T is not​ one-to-one because the standard matrix A has a free variable. D. T is​ one-to-one because the column vectors are not scalar multiples of each other. b. Is the linear transformation​ onto? A. T is onto because the columns of the standard matrix A span . B. T is not onto because the columns of the standard matrix A span . C. T is onto because the standard matrix A does not have a pivot position for every row. D. T is not onto because the standard matrix A contains a row of zeros.

Answers

Answer:

C. T is not​ one-to-one because the standard matrix A has a free variable.

Step-by-step explanation:

Given

[tex]T(x_1,x_2,x_3) = (x_1-5x_2+4x_3,x_2 - 6x_3)[/tex]

Required

Determine if it is linear or onto

Represent the above as a matrix.

[tex]T(x_1,x_2,x_3) = \left[\begin{array}{ccc}1&-5&4\\0&1&-6\\0&0&0\end{array}\right] \left[\begin{array}{c}x_1&x_2&x_3\end{array}\right][/tex]

From the above matrix, we observe that the matrix does not have a pivot in every column.

This means that the column are not linearly independent, & it has a free variable and as such T is not one-on-one

Can somone help me solve this

Answers

Answer:

solid shade above

Step-by-step explanation:

graph line same way you would if it was an equal sign

all the y's that are greater are above the line

its solid because is greater than OR equal to. Equal to includes the line itself as a set of solutions

what is the largest six-digit base five number? How do you know?

Answers

Answer:

999999.9>repeating

Step-by-step explanation:

.1 more and it will be 7 digits and not 6 Hope this helps :D

9514 1404 393

Answer:

  444444₅ = 15624₁₀

Step-by-step explanation:

It will be 1 less than the smallest 7-digit base-5 number. That number is 5^6 = 15625, so the largest 6-digit base 5 number is 15624 in decimal. In base 5, it is 6 of the largest base-5 digits:

  444444₅ = 15624₁₀

__

The digits of the number cannot be any larger than the largest base-5 digit. The base-5 digits are 0 to 4, so the largest is 4.

A rectangular room has a perimeter of 70m what could be the length of the longest side of the room​

Answers

Answer:

34

Step-by-step explanation:

70-2=68

side 1 & 3=1m each

68 divided by 2 = 34

side 2 & 4 = 34m each

I need help plzzz
How can you construct the perpendicular bisector of segment RS by folding a piece of paper

Answers

9514 1404 393

Answer:

  (c)  fold it so R lies on top of S

Step-by-step explanation:

The crease will be the perpendicular bisector of RS when R and S are equidistant from it. That can be accomplished by folding the paper so points R and S lie on top of each other.

kelly bought two dozen apples she found 1/3 of them were rotten how manu apples were rotten

Answers

the answer is 8 apples were rotten

Answer:

8 apples are rotten.

Step-by-step explanation:

2*12=24 . There are 12 apples in a dozen.

24/3=8

For brainlest answer

Answers

Answer:

3..

Step-by-step explanation:

dont know the explanation sry i m beginner so .

Insurance companies need to maintain a certain amount in reserved funds in order to pay anticipated claims. The average monthly claim amount for the last 60 months for company A was $7,500,000 and the (sample) standard deviation was $1,200,000.
(a) Find a 95% upper confidence bound on the average monthly claim amount.
(b) The regulations on the reserves will be strengthened in the near future. According to the new regulations, insurance companies that do not have sufficient amount in reserve will be subject to a significant penalty. Company A wants to adjust the target reserve amount accordingly, by computing a new upper bound on the average monthly claim amount. Should the company recalculate an upper confidence bound with a higher or a lower level of confidence? Briefly explain why. Then compute a 99.95% upper confidence bound on the average monthly claim amount.

Answers

Answer:

a ) Upper bound of CI  is 7803483.87

b) The new upper bound is  8039767.74

Step-by-step explanation:

From sample data:

sample size    n  =  60

sample mean  x = 7500000

Sample standard deviation   s  =  1200000

a) Confidence Interval 95 %  then  significance level  α = 5 % o  α = 0.05

α/2 = 0.025     z(c) from z-table is  z(c) = 1.96

CI  95 % =  (   x  ±  z(c) * s/√n

CI  95 % =  (  7500000  ±  1.96 * 1200000/√60

CI  95 % =  (  7500000  ±  303483.87 )

CI  95 % =  ( 7196516.13  ;  7803483.87)

Then upper bound of CI  is 7803483.87

b) The company has to decrease the significance level equivalent to widen the confidence interval.

If CI now is   99.95 %    significance level is α = 0.0005 and

α/2 = 0.00025     z(c) for that α/2    is from z-table  z(c) ≈ 3.486

CI 99.95 %  = ( x ±  z(c)*s/√n

CI 99.95 %  =  7500000 ±  3.486*1200000/ √60

CI 99.95 %  =  (7500000 ±539767.74)

CI 99.95 %  = ( 6960232.26 ; 8039767.74)

The new upper bound is  8039767.74

f(x)=2x^2-x-6 g(x)=4-x (f+g)(x)

Answers

[tex] \large \boxed{(f + g)(x) = f(x) + g(x)}[/tex]

The property above is distribution property where we distribute x-term in the function.

Substitute both f(x) and g(x) in.

[tex] \large{ \begin{cases} f(x) = 2 {x}^{2} - x - 6 \\ g(x) = 4 - x \end{cases}} \\ \large{f(x) + g(x) = (2 {x}^{2} - x - 6) + (4 - x)} \\ \large{f(x) + g(x) = 2 {x}^{2} - x - 6 + 4 - x}[/tex]

Évaluate/Combine like terms.

[tex] \large{f(x) + g(x) = 2 {x}^{2} - 2x - 2} [/tex]

The function can be factored so there are two answers. (Both of them work as one of them is factored form while the other one is not.)

[tex] \large{(f + g)(x) = 2 {x}^{2} -2x -2}[/tex]

Alternative

[tex] \large{(f + g)(x) = 2({x}^{2} -x - 1)}[/tex]

Answer

(f+g)(x) = 2x²-2x-2(f+g)(x) = 2(x²-x-1)

Both answers work. The second answer is in factored form.

Let me know if you have any doubts!

Choose all the points that would be acceptable points to choose to test for shading for −3(m+7)≥−24

Answers

Answer:

m≤1

Step-by-step explanation:

The given inequality is :

−3(m+7)≥−24

Dividing both sides by -3.

(m+7) ≤ 8

Subtract 7 from both sides,

(m+7)-7 ≤ 8-7

m≤1

Hence, this is the required solution.

)Which statement best describes the area of the triangle shown below?

A coordinate grid is shown with a triangle.. The base is 4 units, and the height is 4 units.

It is one-half the area of a rectangle of length 4 units and width 2 units.
It is one-half the area of a square of side length 4 units.
It is twice the area of a rectangle of length 4 units and width 2 units.
It is twice the area of a square of side length 4 units.

Answers

9514 1404 393

Answer:

  (b)  It is one-half the area of a square of side length 4 units.

Step-by-step explanation:

The triangle described can be formed by drawing the diagonal through a square 4 units on a side. Such a diagonal cuts the square cleanly in half, so the area of the triangle is half the area of the square.

Answer:

B

Step-by-step explanation:

Hope you have a nice day! :o)

Part
The number 3.14 is a rounded value of the mathematical quantity . If you replace 3.14 with 1, what is the exact formula for the circumference in
terms of the diameter, d, and in terms of the radius, r?
B IV x x
Font Sizes
А
E1 E2
Characters used: 0/15000
Submit
I need help

Answers

Answer:

C = 2(3.14)r

C = (3.14)d

Step-by-step explanation:

Circumference in terms of radius:

[tex]c \: = 2\pi \: r[/tex]

So replacing pi with 3.14 you would get 2(3.14)r

Circumference in terms of diameter:

[tex]c = d\pi[/tex]

So replacing pi with 3.14 you would get d(3.14)

which exponential equation is equivalent to the logarithmic equation below? log478=a
A. 478^10 = a
B. a^10 = 478
C. 478^a = 10
D. 10^a = 478

Answers

I think the answer is D

Find the length of arc AC.
B

120°
10
A
С

Answers

Answer:

36.2

Step-by-step

Will Mark Brainlest Help pls ​

Answers

Answer:

13

HOPE THIS HELPS YOU......

If the measure of angle 2 Is (4x+10)degrees and the measure of angle 3 is (3x-5)degrees, what is the measure of angle 2 in degrees?

Answers

Answer:

110°

Step-by-step explanation:

4x+10 + (3x-5) = 180

7x + 5 = 180

7x = 175

x = 25

∠2 = 4(25) + 10 = 110

The measure of angle 2 in degrees, when both lines are perpendicular, is approximately 58.56 degrees.

To find the measure of angle 2 when the lines are perpendicular, we can set up an equation. Since the angles are formed by perpendicular lines, we know that the sum of angle 2 and angle 3 will be equal to 90 degrees. Mathematically, we can write this as:

(4x + 10) + (3x - 5) = 90

Now, let's solve the equation to find the value of x. First, combine like terms:

4x + 3x + 10 - 5 = 90

7x + 5 = 90

Next, isolate x by subtracting 5 from both sides:

7x = 85

Finally, divide by 7 to solve for x:

x = 85 / 7 ≈ 12.14

Now that we have the value of x, we can find the measure of angle 2 by substituting x back into its expression:

Angle 2 = 4x + 10

Angle 2 = 4(12.14) + 10

Angle 2 ≈ 48.56 + 10

Angle 2 ≈ 58.56 degrees

To know more about angle here

https://brainly.com/question/4316040

#SPJ2

for which of the following equations (-4, -3 ) not a solution

A. 2y - 3x = 6
B. -3x = 4y
c. y = 1 + x
D. 5x + 2y = -26​

Answers

Answer: B!

For this you can just plug in, if it’s a true statement it’s a solution.

A. 2(-3)-3(-4)=6
-6+12= 6 that’s a solution.

B. -3(-4) = 4(-3) this is not a solution. 12 does not equal -12

C. -3= 1-4 that’s a solution.

D. 5(-4)+ 2(-3) =-26
-20 -6 = -26 that’s a solution.

The area of a triangle is 2/5 square feet. What is the base of the triangle if the height is 6/5 feet?
24/25 feet
25/24 feet
2/3 feet
3/2 feet​

Answers

Answer:

2/3 feet.

Step-by-step explanation:

The formula for finding the area of a triangle is  A = (B x H)/2. Or, area is equal to base times height, all divided by two. In your case, we don't know the base. If we substitute in values, we get 2/5 = (B x 6/5)/2.

Next, we have to multiply both sides by two, to cancel the "/2." That results in 4/5 = B x 6/5. Then, to isolate the variable, we have to divide both sides by 6/5. 4/5 ÷ 6/5 is 4/5 x 5/6. The fives cancel out, and we are left with 4/6.

Finally, we simplify! 4/6 simplifies to 2/3.

What is the MEDIAN or Q2 of the data set: 24, 25, 29, 30, 31, 31, 32, 34, 34? ​

Answers

Answer:

31

Step-by-step explanation:

Median = middle number in terms of value

Usually the first step is to list the numbers in order from least to greatest however the numbers are already listed in order from least to greatest

24, 25, 29, 30, 31, 31, 32, 34, 34

We the just go to the middle number

The middle number would be 31

So we can conclude that the median is 31

Use the listing method to write the following set \ 1,2,3\; [0, 1, 2, 3] O (1, 2)

Answers

0,1 is everything but you have to be 3/2 to know what your doing

A mouse is trapped in a maze. To
find his way out, he walks 15 m,
makes a 90° left turn, walks 8 m,
makes another 90° left turn, and
walks 10 m. What is the magnitude
of the resultant vector?

Answers

Answer: Approximately 9.43 meters

The more accurate version is 9.4339811320566 but that's not exact.

This is exact magnitude is sqrt(89)

=======================================================

Explanation:

Let's draw an xy grid for this. Place the starting point at (0,0) which I'll call point A. Now let's say the mouse goes east 15 meters. That would move it to (15,0) which is marked as point B. Refer to the diagram below.

From point B, we move to C which is at (15,8). So the mouse has gone 8 meters north. It might help to turn the page so that the east direction is facing completely north, and then look to the left and you'll see "north". In other words, each 90 degree left turn is a 90 degree counterclockwise turn.

After doing another 90 degree counterclockwise turn, the mouse will move 10 meters westward. It moves from C(15,8) to D(5,8). Point D is the final position of the mouse.

-----------------------

A(0,0) was the initial position and D(5,8) is the final position

The vector v is

v = <5,8>

This is because we basically are saying "the mouse ultimately moved 5 meters east and 8 meters north" when going from start to finish. We're ignoring the intermediate stops along the way.

------------------------

Recall that for any vector of the form

v = <a,b>

the magnitude of that vector is

|v| = sqrt(a^2+b^2)

this is the length of the vector based on the pythagorean theorem.

Applying this formula gets us

|v| = sqrt(5^2+8^2)

|v| = sqrt(89)

|v| = 9.43398 approximate

This represents the straight line distance from start to finish, where we ignore any intermediate stops. So this isn't the distance the mouse travels (since it goes from A to B, to C to D). Instead, it's the distance it would travel if it wanted to take the shortest path from A to D.

Answer:

Step-by-step explanation:

9.4 for acellus

2x^2 + 3x - 12 when x = 5 help pls

Answers

2x^2 + 3(5)-12
2x^2+ 15+-12

combine like terms

2x^2+15+-12
(2x^2)+(15+-12)

Answer:
2x^2 + 3

[tex]\huge\textsf{Hey there!}[/tex]

[tex]\mathsf{2x^2 + 3x - 12}[/tex]

[tex]\mathsf{= 2(5)^2 + 3(5) - 12}[/tex]

[tex]\mathsf{5^2}[/tex]

[tex]\mathsf{= 5 \times 5}[/tex]

[tex]\mathsf{\bf = 25}[/tex]

[tex]\mathsf{2(25) + 3(5) - 12}[/tex]

[tex]\mathsf{2(25)}[/tex]

[tex]\mathsf{= \bf 50}[/tex]

[tex]\mathsf{3(5)}[/tex]

[tex]\mathsf{= \bf 15}[/tex]

[tex]\mathsf{= 50 + 15 - 12}[/tex]

[tex]\mathsf{50 + 15}[/tex]

[tex]\mathsf{= \bf 65}[/tex]

65 - 12

[tex]\mathsf{= \bf 53}[/tex]

[tex]\boxed{\boxed{\huge\textsf{Answer: \bf 53}}}\huge\checkmark[/tex]

[tex]\large\textsf{Good luck on your assignment and enjoy your day!}[/tex]

~[tex]\frak{Amphitrite1040:)}[/tex]

ANSWER ASAP!! Use the substitution method to solve the system of equations. 3x+7y=1 and y=x-7

Answers

Answer:

x=5

y = -2

(5,-2)

Step-by-step explanation:

3x+7y=1 and y=x-7

Substitute the second equation in for y in the first equation

3x+7( x-7)=1

Distribute

3x+ 7x - 49 = 1

Combine like terms

10x - 49 =1

Add 49 to each side

10x-49+49 = 1+49

10x = 50

Divide by 10

10x/10 = 50/10

x = 5

Now find y

y = x-7

y = 5-7

y = -2

The base of a triangle is 9cm correct to the nearest cm.
The area of this triangle is 40 cm2 correct to the nearest 5 cm?.
Calculate the upper bound for the perpendicular height of this triangle.

Answers

Answer:

can you show the picture plz

What is the slope of a line perpendicular to the line whose equation is x + 2y = 2. Fully simplify your answer. ​

Answers

9514 1404 393

Answer:

  2

Step-by-step explanation:

We can solve for y to find the slope of the given line.

  x +2y = 2

  2y = -x +2 . . . . . subtract x

  y = -1/2x +1 . . . . divide by the coefficient of y

The slope is the x-coefficient, -1/2.

Any perpendicular line will have a slope that is the opposite reciprocal of this:

  -1/(-1/2) = 2

A perpendicular line will have a slope of 2.

Consider the following equation. -2x + 6 =[-2/3] + 5

Answers

Answer:

that's my answer

hope it helps

Plz help me solve this algebra

Answers

Answer:

Step-by-step explanation:

[tex]16^{\frac{24}{64}} =16^{\frac{4*2*3}{4*2*8}}\\\\=(2^{4})^{\frac{3}{8}}\\\\=2^{4*\frac{3}{8}}\\\\=2^{\frac{3}{2}}=2^{3*\frac{1}{2}}\\\\=\sqrt{2*2*2}\\\\=2\sqrt{2}[/tex]

Solve for X.

A. 10.96
B. 4.88
C. 0.04
D. 5.34

Answers

Answer:

4.88

Step-by-step explanation:

That may be the question

1. Decide whether the statement below is true or false. Then fully explain why. The triangle below can b used to help with your explanation if needed.

"The sine of any acute angle is equal to the cosine of its complementary angle."[help me plzz!!]​

Answers

Answer: True

==================================

Work Shown:

sin(angle) = opposite/hypotenuse

sin(alpha) = MN/NP

sin(alpha) = y/z

---------

cos(angle) = adjacent/hypotenuse

cos(beta) = MN/NP

cos(beta) = y/z

----------

Both sin(alpha) and cos(beta) are equal to y/z, so that means sin(alpha) = cos(beta). This is true because alpha+beta = 90, ie the angles are complementary.

Similarly, you should find that sin(beta) and cos(alpha) are both equal to x/z, which is further proof that the statement is true.

1;3;5;7…(determine the value of the 19th term of the sequence)​

Answers

Answer:

37

Step-by-step explanation:

fistly find how much different

3-1=2 5-3=2 7-5=2

so for the next and next value just plus two until the 19th then you will get 37

Answer:    37

=============================================================

Explanation:

a = 1 is the first term

d = 2 is the common difference, since we add 2 to each term to get the next term.

The nth term is a+d(n-1) which becomes 1+2(n-1). That simplifies to 2n-1

The last step is to plug in n = 19 and you should get 2(19)-1 = 37

Other Questions
How does the FITT principle apply to the development of a successful personal fitness program? What government agency sets safety standards for all common household and recreational products ? Explain what the graph of a polynomial that has a degree of 4, zeros of -1, 7, 10,and a leading coefficient that is negative would look like. The earth is surrounded by a _______________________ of gases called the ________________. The atmosphere is very _________________________ to life on ________________ and does many ______________________ to help protect life and help _____________________ to survive. The atmosphere absorbs the ______________ from the _________________ and keeps the heat ________________ the atmosphere helping the _______________________ to stay warm, called the ___________________ Effect. ? PLZ HELP, IT WOULD MEAN THE WORLDA television was originally priced at $522, but Sam waited to buy it until the television was on sale for 50% off. Ifhe paid 8% sales tax on the sale price, how much did he pay in total? write a quadratic function f whose zeros are 2 and -6 should the conowingo dam be removed in Maryland. if yes or no explain Answer to the question What is the difference between inductive reasoning and deductive reasoning? Frank kept track of the amount of money he earned each day for 2 weeks.the amounts,in dollars,are listed below. 25,30,24,20,20,22.5,75,27,27,22,22,27,22.5,28Find and calculate the measures of center and variability that best summarize frank's data.explain Which equation can be used to solve for x in the parallelogram below? 16 km14 kmAn ambulance traveled on roads from the hospital14 kilometers east, then 16 kilometers north toreach an accident. If the ambulance traveled for0.40 hours, what was its average speed?12 kilometers/hourO 30 kilometers/hourO 75 kilometers/hour10 kilometers/hour How does thecartoonistjustify hisIdecision toappease Hitler? Circle D is inscribed in AABC as shown.Point E marks the intersection of the twoshapes. Which of the following statements aretrue?Statement I: DE is the radius of theinscribed circle.Statement II: DE is perpendicular to AC.Statement III: Point D is equidistant fromvertices A, B, and C. Which set of numbers can represent the side lengths, in inches, of an acute triangle? 4, 5, 7 5, 7, 8 6, 7, 10 7, 9, 12 Which set of numbers can represent the side lengths, in centimeters, of a right triangle? 8, 12, 15 10, 24, 26 12, 20, 25 15, 18, 20 An object falls from rest at the top of a building. If it falls 0.3 of the height of the building in the first 13 seconds of motion, find the total time of its fall in seconds. If a security of $10,000 will be worth $15,036.30 seven years in the future, assuming that no additional deposits or withdrawals are made, what is the implied interest rate the investor will earn on the security De acuerdo a la primera estrofa del poema 'Caupolicn' de Rubn Daro, cmo te imaginas a Caupolicn? Realiza una descripcin de acuerdo a lo planteado en la estrofa. If you travel 40 m South and 100 m North, what is your displacement? A file that is 226 megabytes is being downloaded if the download is 12.&% complete, how many megabytes have been downloaded? Round your answer to the nearest tenth